Difference between revisions of "1986 AJHSME Problems/Problem 6"
5849206328x (talk | contribs) m (→Solution) |
|||
Line 7: | Line 7: | ||
==Solution== | ==Solution== | ||
− | + | Just simplify the bottom as <math>\frac{3}{3}-\frac{2}{3}=\frac{1}{3}</math>, getting us <math>\frac{2}{\frac{1}{3}}</math>, with which we multiply top and bottom by 3, we get <math>\frac{6}{1}</math>, or <math>6</math> | |
− | + | <math>\boxed{\text{E}}</math> | |
==See Also== | ==See Also== | ||
[[1986 AJHSME Problems]] | [[1986 AJHSME Problems]] |
Revision as of 09:26, 24 January 2009
Problem
Solution
Just simplify the bottom as , getting us , with which we multiply top and bottom by 3, we get , or